Perfect matching in random bipartite graph - with fixed probability












2












$begingroup$


as a follow up from this question :



Suppose that we have a simpler problem, where the probability $p$ is fixed. Of course we could use the above result to proove that almost every graph in the model $mathcal{G}_{n,n,p}$ contains a perfect matching, saying for instance that $$ p > frac{sqrt{n} + log{n}}{n}$$ for big enough $n$.



However, couldn't we find a more direct result?
I've been looking at the probability of findind a set $S$ (size $k$) giving a contradiction for Hall's mariage theorem. With $q=1-p$, we need each element of $S$ not connected to the $n-k-1$ elements not in $N(S)$, therefore :
$$ Pr[ |N(S)|<|S| ] leq q^{nk-k(k-1)}$$



And
$$Pr[nexists text{ a perfect matching}] leq sum_{k=1}^n binom{n}{k}q^{nk-k(k-1)}$$



But from there, trying to proove that this sum tends to 0, simple binomial approximation $sim n^k/k!$ gives too large a bound. Would you recommand any other calculation?
Thanks










share|cite|improve this question









$endgroup$

















    2












    $begingroup$


    as a follow up from this question :



    Suppose that we have a simpler problem, where the probability $p$ is fixed. Of course we could use the above result to proove that almost every graph in the model $mathcal{G}_{n,n,p}$ contains a perfect matching, saying for instance that $$ p > frac{sqrt{n} + log{n}}{n}$$ for big enough $n$.



    However, couldn't we find a more direct result?
    I've been looking at the probability of findind a set $S$ (size $k$) giving a contradiction for Hall's mariage theorem. With $q=1-p$, we need each element of $S$ not connected to the $n-k-1$ elements not in $N(S)$, therefore :
    $$ Pr[ |N(S)|<|S| ] leq q^{nk-k(k-1)}$$



    And
    $$Pr[nexists text{ a perfect matching}] leq sum_{k=1}^n binom{n}{k}q^{nk-k(k-1)}$$



    But from there, trying to proove that this sum tends to 0, simple binomial approximation $sim n^k/k!$ gives too large a bound. Would you recommand any other calculation?
    Thanks










    share|cite|improve this question









    $endgroup$















      2












      2








      2





      $begingroup$


      as a follow up from this question :



      Suppose that we have a simpler problem, where the probability $p$ is fixed. Of course we could use the above result to proove that almost every graph in the model $mathcal{G}_{n,n,p}$ contains a perfect matching, saying for instance that $$ p > frac{sqrt{n} + log{n}}{n}$$ for big enough $n$.



      However, couldn't we find a more direct result?
      I've been looking at the probability of findind a set $S$ (size $k$) giving a contradiction for Hall's mariage theorem. With $q=1-p$, we need each element of $S$ not connected to the $n-k-1$ elements not in $N(S)$, therefore :
      $$ Pr[ |N(S)|<|S| ] leq q^{nk-k(k-1)}$$



      And
      $$Pr[nexists text{ a perfect matching}] leq sum_{k=1}^n binom{n}{k}q^{nk-k(k-1)}$$



      But from there, trying to proove that this sum tends to 0, simple binomial approximation $sim n^k/k!$ gives too large a bound. Would you recommand any other calculation?
      Thanks










      share|cite|improve this question









      $endgroup$




      as a follow up from this question :



      Suppose that we have a simpler problem, where the probability $p$ is fixed. Of course we could use the above result to proove that almost every graph in the model $mathcal{G}_{n,n,p}$ contains a perfect matching, saying for instance that $$ p > frac{sqrt{n} + log{n}}{n}$$ for big enough $n$.



      However, couldn't we find a more direct result?
      I've been looking at the probability of findind a set $S$ (size $k$) giving a contradiction for Hall's mariage theorem. With $q=1-p$, we need each element of $S$ not connected to the $n-k-1$ elements not in $N(S)$, therefore :
      $$ Pr[ |N(S)|<|S| ] leq q^{nk-k(k-1)}$$



      And
      $$Pr[nexists text{ a perfect matching}] leq sum_{k=1}^n binom{n}{k}q^{nk-k(k-1)}$$



      But from there, trying to proove that this sum tends to 0, simple binomial approximation $sim n^k/k!$ gives too large a bound. Would you recommand any other calculation?
      Thanks







      graph-theory random-graphs matching-theory






      share|cite|improve this question













      share|cite|improve this question











      share|cite|improve this question




      share|cite|improve this question










      asked Dec 11 '18 at 10:03









      Thomas LesgourguesThomas Lesgourgues

      845117




      845117






















          1 Answer
          1






          active

          oldest

          votes


















          1












          $begingroup$

          Many approximations to the binomial $binom nk$ are not symmetric in $k$ and $n-k$, so they perform badly when $k$ is close to $n$. (For example, $frac{n^k}{k!}$ is somewhere around $e^n$ when $k=n$, even though actually $binom nn = 1$.)



          Here, the sum is almost but not quite symmetric: we can write the power of $q$ as $q^{k(n+1-k)} < q^{k(n-k)}$, and the latter is symmetric. So we can bound the sum by
          $$
          2 sum_{k=1}^{n/2} binom nk q^{k(n-k)}
          $$

          and avoid the problematic values of $k$, getting a sum that's easy to bound using your approximation. It's even enough to write $binom nk le n^k$ and get $binom nk q^{k(n-k)} le (n cdot q^{n-k})^k le (n cdot q^{n/2})^k$.






          share|cite|improve this answer











          $endgroup$













            Your Answer





            StackExchange.ifUsing("editor", function () {
            return StackExchange.using("mathjaxEditing", function () {
            StackExchange.MarkdownEditor.creationCallbacks.add(function (editor, postfix) {
            StackExchange.mathjaxEditing.prepareWmdForMathJax(editor, postfix, [["$", "$"], ["\\(","\\)"]]);
            });
            });
            }, "mathjax-editing");

            StackExchange.ready(function() {
            var channelOptions = {
            tags: "".split(" "),
            id: "69"
            };
            initTagRenderer("".split(" "), "".split(" "), channelOptions);

            StackExchange.using("externalEditor", function() {
            // Have to fire editor after snippets, if snippets enabled
            if (StackExchange.settings.snippets.snippetsEnabled) {
            StackExchange.using("snippets", function() {
            createEditor();
            });
            }
            else {
            createEditor();
            }
            });

            function createEditor() {
            StackExchange.prepareEditor({
            heartbeatType: 'answer',
            autoActivateHeartbeat: false,
            convertImagesToLinks: true,
            noModals: true,
            showLowRepImageUploadWarning: true,
            reputationToPostImages: 10,
            bindNavPrevention: true,
            postfix: "",
            imageUploader: {
            brandingHtml: "Powered by u003ca class="icon-imgur-white" href="https://imgur.com/"u003eu003c/au003e",
            contentPolicyHtml: "User contributions licensed under u003ca href="https://creativecommons.org/licenses/by-sa/3.0/"u003ecc by-sa 3.0 with attribution requiredu003c/au003e u003ca href="https://stackoverflow.com/legal/content-policy"u003e(content policy)u003c/au003e",
            allowUrls: true
            },
            noCode: true, onDemand: true,
            discardSelector: ".discard-answer"
            ,immediatelyShowMarkdownHelp:true
            });


            }
            });














            draft saved

            draft discarded


















            StackExchange.ready(
            function () {
            StackExchange.openid.initPostLogin('.new-post-login', 'https%3a%2f%2fmath.stackexchange.com%2fquestions%2f3035133%2fperfect-matching-in-random-bipartite-graph-with-fixed-probability%23new-answer', 'question_page');
            }
            );

            Post as a guest















            Required, but never shown

























            1 Answer
            1






            active

            oldest

            votes








            1 Answer
            1






            active

            oldest

            votes









            active

            oldest

            votes






            active

            oldest

            votes









            1












            $begingroup$

            Many approximations to the binomial $binom nk$ are not symmetric in $k$ and $n-k$, so they perform badly when $k$ is close to $n$. (For example, $frac{n^k}{k!}$ is somewhere around $e^n$ when $k=n$, even though actually $binom nn = 1$.)



            Here, the sum is almost but not quite symmetric: we can write the power of $q$ as $q^{k(n+1-k)} < q^{k(n-k)}$, and the latter is symmetric. So we can bound the sum by
            $$
            2 sum_{k=1}^{n/2} binom nk q^{k(n-k)}
            $$

            and avoid the problematic values of $k$, getting a sum that's easy to bound using your approximation. It's even enough to write $binom nk le n^k$ and get $binom nk q^{k(n-k)} le (n cdot q^{n-k})^k le (n cdot q^{n/2})^k$.






            share|cite|improve this answer











            $endgroup$


















              1












              $begingroup$

              Many approximations to the binomial $binom nk$ are not symmetric in $k$ and $n-k$, so they perform badly when $k$ is close to $n$. (For example, $frac{n^k}{k!}$ is somewhere around $e^n$ when $k=n$, even though actually $binom nn = 1$.)



              Here, the sum is almost but not quite symmetric: we can write the power of $q$ as $q^{k(n+1-k)} < q^{k(n-k)}$, and the latter is symmetric. So we can bound the sum by
              $$
              2 sum_{k=1}^{n/2} binom nk q^{k(n-k)}
              $$

              and avoid the problematic values of $k$, getting a sum that's easy to bound using your approximation. It's even enough to write $binom nk le n^k$ and get $binom nk q^{k(n-k)} le (n cdot q^{n-k})^k le (n cdot q^{n/2})^k$.






              share|cite|improve this answer











              $endgroup$
















                1












                1








                1





                $begingroup$

                Many approximations to the binomial $binom nk$ are not symmetric in $k$ and $n-k$, so they perform badly when $k$ is close to $n$. (For example, $frac{n^k}{k!}$ is somewhere around $e^n$ when $k=n$, even though actually $binom nn = 1$.)



                Here, the sum is almost but not quite symmetric: we can write the power of $q$ as $q^{k(n+1-k)} < q^{k(n-k)}$, and the latter is symmetric. So we can bound the sum by
                $$
                2 sum_{k=1}^{n/2} binom nk q^{k(n-k)}
                $$

                and avoid the problematic values of $k$, getting a sum that's easy to bound using your approximation. It's even enough to write $binom nk le n^k$ and get $binom nk q^{k(n-k)} le (n cdot q^{n-k})^k le (n cdot q^{n/2})^k$.






                share|cite|improve this answer











                $endgroup$



                Many approximations to the binomial $binom nk$ are not symmetric in $k$ and $n-k$, so they perform badly when $k$ is close to $n$. (For example, $frac{n^k}{k!}$ is somewhere around $e^n$ when $k=n$, even though actually $binom nn = 1$.)



                Here, the sum is almost but not quite symmetric: we can write the power of $q$ as $q^{k(n+1-k)} < q^{k(n-k)}$, and the latter is symmetric. So we can bound the sum by
                $$
                2 sum_{k=1}^{n/2} binom nk q^{k(n-k)}
                $$

                and avoid the problematic values of $k$, getting a sum that's easy to bound using your approximation. It's even enough to write $binom nk le n^k$ and get $binom nk q^{k(n-k)} le (n cdot q^{n-k})^k le (n cdot q^{n/2})^k$.







                share|cite|improve this answer














                share|cite|improve this answer



                share|cite|improve this answer








                edited Dec 11 '18 at 22:28

























                answered Dec 11 '18 at 22:22









                Misha LavrovMisha Lavrov

                46.9k657107




                46.9k657107






























                    draft saved

                    draft discarded




















































                    Thanks for contributing an answer to Mathematics Stack Exchange!


                    • Please be sure to answer the question. Provide details and share your research!

                    But avoid



                    • Asking for help, clarification, or responding to other answers.

                    • Making statements based on opinion; back them up with references or personal experience.


                    Use MathJax to format equations. MathJax reference.


                    To learn more, see our tips on writing great answers.




                    draft saved


                    draft discarded














                    StackExchange.ready(
                    function () {
                    StackExchange.openid.initPostLogin('.new-post-login', 'https%3a%2f%2fmath.stackexchange.com%2fquestions%2f3035133%2fperfect-matching-in-random-bipartite-graph-with-fixed-probability%23new-answer', 'question_page');
                    }
                    );

                    Post as a guest















                    Required, but never shown





















































                    Required, but never shown














                    Required, but never shown












                    Required, but never shown







                    Required, but never shown

































                    Required, but never shown














                    Required, but never shown












                    Required, but never shown







                    Required, but never shown







                    Popular posts from this blog

                    Plaza Victoria

                    In PowerPoint, is there a keyboard shortcut for bulleted / numbered list?

                    How to put 3 figures in Latex with 2 figures side by side and 1 below these side by side images but in...